Jump to content
Urch Forums

debarshi

1st Level
  • Posts

    218
  • Joined

  • Days Won

    1

debarshi last won the day on November 24 2006

debarshi had the most liked content!

Converted

  • My Tests
    No

debarshi's Achievements

Newbie

Newbie (1/14)

5

Reputation

  1. you are correct, only that i would like to correct case 3 xx=3 so the basic idea is |a|=a if a>=0 and |a|=-a if a here a on the LHS is x+1 and RHS is x-1 the final result remains same, thanks
  2. Is ½x½ (1) ½x + 1½ = 2½x - 1½ (2) ½x - 3½≠ 0 oa Can you please explain?
  3. If X, Y>0, If X^3 = Y, is Y a fraction? (1) X^2 is a fraction. (2) X > Y. Can you please discuss? Is 2^(2/3) a fraction?
  4. My reasoning is 0 is an even integer, it is never said a>0 in stm 1; along with stm 2 which establishes a0 , stm1 can be used.
  5. Is a*b*c divisible by 24 ? 1.a, b, and c are consecutive even integers 2.a*b is divisible by 12 Can you please discuss? OA given is , however I think it should be .
  6. If n is a positive integer, is 10^n-1 divisible by q? 1.q is not divisible by either 5 or 2 2.q is not divisible by 9 oa is , however i think it should be B, can you discuss please?
  7. Guys, this is already discussed. However I want somebody please help me undertstanding the differences between A and E. A says, if a tax payer sets aside some part of his borrowed money, on which he gets a tax relief, he will not get any more tax relief on that amount because the savings interest is taxable. My question is how is it exactly NOT helping tax department to get more taxes. My understading is if you borrow $100 and then put that in savings account then tax department will be taxing on the interest of the deposit in addition to the fact they don't have to give a tax relief. Is it not strenghthening the very idea that it is actually helping the tax department? Now E says if you borrow $100, then you can actually pay the taxes that are charged on the interest acrrued on the savings account. So this is against the interest of the tax department. Where am I making a mistake ? Which of the following, if true, is the most logical completion of the argument below? The tax system of the Republic of Grootland encourages borrowing by granting its taxpayers tax relief for interest paid on loans. The system also discourages saving by taxing any interest earned on savings. Nevertheless, it is clear that Grootland’s tax system does not consistently favor borrowing over saving, for if it did, there would be no______ (A) tax relief in Grootland for those portions of a taxpayer’s income, if any, that are set aside to increase that taxpayer’s total savings (B) tax relief in Grootland for the processing fees that taxpayers pay to lending institutions when obtaining certain kinds of loans © tax relief in Grootland for interest that taxpayers are charged on the unpaid balance in credit card accounts (D) taxes due in Grootland on the cash value of gifts received by taxpayers from banks trying to encourage people to open savings account (E) taxes due in Grootland on the amount that a taxpayer has invested in interest-bearing savings accounts
  8. OA is given E. I also picked up D. OA must be wrong, it is from gmat sc prep test 4 Q19.
  9. 19. Unlike modern feminism, which derives much of its strength from rejection of an oppressive home, Marietta Holley in her novels made domesticity the basis of women’s liberation. (A) Unlike modern feminism, which derives much of its strength from rejection of an oppressive home, Marietta Holley in her novels (B) Unlike the modern feminist who derives much of her strength from rejection of an oppressive home, Marietta Holley’ s novels © Unlike modern feminism, deriving much of its strength from rejection of an oppressive home, Marietta Holley’s novels (D) Unlike those of the modern feminist who derives much of her strength from rejection of an oppressive home, novels of Marietta Holley (E) Unlike those of modern feminism, which derive much of their strength from rejection of an oppressive home, Marietta Holley’ s novels This should be an easy one, however I cannot decide between D and E. Can anybody please explain?
  10. Between 1975 and 1985, nursing-home occupancy rates averaged 87 percent of capacity, while admission rates remained constant, at an average of 95 admissions per 1,000 beds per year. Between 1985 and 1988, however, occupancy rates rose to an average of 92 percent of capacity, while admission rates declined to 81 per 1,000 beds per year. If the statements above are true, which of the following conclusions can be most properly drawn? (A) The average length of time nursing-home residents stayed in nursing homes increased between 1985 and 1988. (B) The proportion of older people living in nursing homes was greater in 1988 than in 1975. © Nursing home admission rates tend to decline whenever occupancy rates rise. (D) Nursing homes built prior to 1985 generally had fewer beds than did nursing homes built between 1985 and 1988. (E) The more beds a nursing home has, the higher its occupancy rate is likely to be. oa is , however i choose c. Now to me, both (a) and (d) can be reasons why occupancy rates after 1985 increased, but they don't say why admission rate decreased. For eg. if I suppose (a) to be true, then i have to assume before 1985 people used to admit in nursing-home more frequently and after 1985 , because they stay longer in nursing-home, the rate decreased with the assumption that the number of people visited before and after 1985 is same. Is it not an assumption not supported in the argument? To me conclusion should be c, wich just states what is observed. So what is wrong with it? Please discuss your logic.
  11. yes oa is B. Sorry , I can see now why it should be the choice over d.
  12. According to a recent publication by the city council, the most cost-effective solution to the problem of congestion on State Highway 1 is to build an elevated roadway above the existing six lanes. This solution makes the most sense because it will save the expense of surveying and breaking new ground for an alternate route, and will also mean that crews will need to construct half as many lanes, since the current lanes will remain in use after the elevated portion is completed. The conclusion in the passage above is based on which of the following assumptions? a.That the city council has already approved funding for an elevated roadway above the existing highway. b.That the cost of surveying and breaking new ground for an alternate route is greater than the cost of constructing an elevated roadway. c.That the large number of people traveling State Highway 1 every day make easing congestion a priority for the city council. d.That the city council can not afford to fund the construction of more than six lanes, and so seeks a solution that will require building as few lanes as possible. e.That drivers on State Highway 1 are unwilling to pay a toll to use the road, and so the city council must provide all funds for improvement. Guys, can you please discuss between b and d? Thanks in advance, i am seeing a flaw in b specially, want to check with some fresh minds :-)
  13. Can you please discuss this ? What is wrong with C? Why painting the face and perfuming the body should have persisted throughout recorded history has been the subject of a substantial amount of anthropological and psychological speculation. (A) Why painting the face and perfuming the body should have persisted (B) How painting the face and perfuming the body could be persistent © That painting the face and perfuming the body are persisting (D) Painting the face and perfuming the body persisting (E) The persisting of face painting and body perfuming OA
  14. I think you are right. "Though" and "But" cannot go in the same sentence. It must be "have thought".
×
×
  • Create New...